Você está na página 1de 33

Page 1 of 33

otherwise, it would unlawful and unconstitutional and the seized


article would be inadmissible evidence.
 In the case at bar, Accused-appellants were not being arrested at the
PEOPLE OF THE PHILIPPINES, time that the subject articles were allegedly taken from them but were
vs. just being questioned by the police officers conducting the
CRISANTO BAULA, RUBEN BAULA, ROBERT investigation about the death of Patrocinia Caburao. The investigating
BAULA and DANILO DACUCOS, officers had no personal knowledge of facts indicating that the accused
had committed the crime. Being in no position to effect a warrantless
Facts: arrest, the police officers were thus likewise barred from effecting a
warrantless search and seizure.
 After the gruesome killing of Patronicia Caburao, the investigating  An illegal search cannot be undertaken and then an arrest effected
police went to the residence of the accused-appellant, Baula et al. on the strength of the evidence yielded by that search.
 In the process of questioning the appellants, the police saw  The Court finds it less than credible the stance of the prosecution that
bloodstained bolo, short pants, polo shirts and was the polo shirt and short pants have been voluntarily given. An alleged
subsequently confiscated without search warrant and directed to the consent to a warrantless search and seizure cannot be based merely on
NBI for forensic exams. The exam resulted that the bloods found in the presumption of regularity in the performance of duty. This
the confiscated articles bears the same blood type “O” as that of the presumption, by itself, cannot prevail against the constitutionally
victim. protected rights of an individual, and zeal in the pursuit of criminals
 Thus, the accused were arrested, charged and was convicted in the cannot enable the use of arbitrary methods that the Constitution itself
crime of murder by the RTC Lingayen and sentenced to suffer abhors.
Reclusion Perpetua. Hence this appeal for review on the decision of  Thus, the bloodstained polo, bolo and shorts are inadmissible as
the lower court in the ground that the articles sought (bloodstained evidence.
bolo, shirt and short pants) cannot be admitted as evidence against the
accused since it was seized without a valid search and seizure warrant.

Issue:
Whether or not the warrantless search conducted was valid under a consented SPOUSES VEROY VS LAYAGUE
search G.R. No. L-95630, 210 SCRA 97, June 18, 1992

Held: NO "Qualified consent"

 The articles are unlawfully searched and seized. The permission to enter a house and search for persons and effects may be
 A search incidental to a valid arrest is one of the statutory exceptions qualified, and the searching officer may not act in excess of the authority
to the constitutional mandate that no search and seizure shall be granted to him.
effected without a valid warrant. In this instance, the arrest should be
lawful before search and seizure by the arresting officer would be Although the offense of illegal possession of firearms is a malum prohibitum,
conducted. A warrantless arrest may be effected by the arresting it does not follow that the subjects may be seized simply because they are
officer when in his presence the person arrested have committed, prohibited. A search warrant is still necessary in the context of this case.
committing or attempting to commit the crime. It cannot be reversed;
Page 2 of 33

 The Court ruled that the case at bar does not fall on the exceptions for
Facts: a warrantless search. The reason for searching the house is that it was
 The Veroys moved to QC and left their house in Davao City to a reportedly being used as a hideout and recruitment center for rebel
caretaker who had keys to the kitchen only. The Veroys had the keys soldiers. While Capt. Obrero was able to enter the yard, he did not
to the interior of the house. enter the house because he did not have a search warrant and the
 Capt. Obrero raided the house based on an information that rebel owners were not present. This shows that he himself recognized the
soldiers are allegedly hiding there. need for a search warrant, hence, he did not persist in entering the
 With the help of caretakers, they were able to enter only up to the yard house but rather contacted the Veroys to seek permission to enter the
since the owner was not around and they did not have a search warrant. same. Permission was granted by Mrs. Veroy to enter the house but
 They contacted Mrs. Veroy, and explained that the house was only to ascertain the presence of rebel soldiers.
reportedly being used as a hideout and recruitment center of rebel  Under the circumstances the police officers had time to procure a
soldiers. Mrs. Veroy then gave permission to search the house with search warrant but they did not.
the condition that Major Macasaet, a long-time family friend, must be  The Court also ruled that although the offense of illegal possession of
there during the search. firearms is a malum prohibitum, it does not follow that the subjects
 Despite the qualified consent, the officers entered various rooms, may be seized simply because they are prohibited. A search warrant is
including the children’s room, and confiscated a .45 caliber gun and still necessary.
other effects, which were the basis of the charge of illegal possession  The rule having been violated and no exception being applicable, the
of firearms against them. articles seized were confiscated illegally and are therefore protected
 Despite the fact that the warrants for their arrest have not yet been by the exclusionary principle. They cannot be used as evidence against
served on them, petitioners voluntarily surrendered themselves to the petitioners in the criminal action against them for illegal
Brig. Gen. Pantaleon Dumlao, PC-CIS Chief, since it was the CIS that possession of firearms.
initiated the complaint. However, the latter refused to receive them on
the ground that his office has not yet received copies of their warrants
of arrest. PASSION VDA. DE GARCIA VS LOCSIN
 The Spouses Veroy assailed the admissibility of the evidence for being
obtained in violation of their constitutional right against unreasonable De Garcia’s person, house or store were subjected to a search by virtue of a
search and seizure. search warrant.Said search was performed, together with the PC, by the agent
of the Anti-Usury Board. Due to the confinement of petitioner due to an illness,
Issue: Whether the evidence is admissible? NO. the agent showed the search warrant to petitioner’s bookkeeper. Seized were
two packages of records and a locked -filing cabinet containing several papers
 Petitioners alleged that while Capt. Obrero had permission to enter
and documents
their house, it was merely for the purpose of ascertaining the presence
of the alleged "rebel" soldiers. The permission did not include the Facts:
authority to conduct a room to room search inside the house. The items
taken were, therefore, products of an illegal search, violative of their  An agent of the Anti-Usury Board (Mariano Almeda) obtained from
constitutional rights. As such, they are inadmissible in evidence
the justice ofpeace of Tarlac a search warrant commanding any officer
against them.
of the law to search the person, house or store of petitioner at Victoria,
Page 3 of 33

Tarlac for “certain books, lists,chits, receipts, documents and other actual or constructive, of the existence of such right, and (c) said person had
papers relating to her activities as usurer.” an actual intention to relinquish said right. The constitutional immunity from
 On the same date (November 10, 1934), Almeda, together with the unreasonable searches and seizures, being a personal one, cannot be waived
captain of the Philippine Constabulary, went to petitioner’s office in by anyone except the person whose rights are invaded or one who is expressly
Victoria, Tarlac. After showing the search warrant to petitioner’s authorized to do so in his/her behalf. In the case at bar, she could not have
bookkeeper (Alfredo Salas), and without the presence of petitioner objected because she was sick and was not present when the warrant was
who was ill and confined at the time, Almeda proceeded with the served upon. Moreover, upon knowing of the seizure of some of her documents
warrant’s execution. Two packages of records and a locked filing and papers, she had sent her lawyers to the office of the Anti-Usury Board to
cabinet containing several papers and documents were seized. Said demand the return of the documents seized. The failure on the part of the
papers and documents were kept for a considerable length of time by petitioner and her bookkeeper to resist or object to the execution of the warrant
the Anti-Usury Board and were turned over by it (the Board) to the does not constitute an implied
fiscal who filed 6 separate criminal cases against petitioner for
violation of the Anti-Usury Law.
 After the seizure, petitioner demanded the return of the documents JOSE G. LOPEZ
seized.Moreover, the legality of the search warrant was challenged by vs.
the petitioner twice(January 7 and June 4, 1937) THE COMMISSIONER OF CUSTOMS, REPARATIONS COMMISSION,
DIRECTOR OF THENATIONAL BUREAU OF INVESTIGATION AND/OR
Ruling: ANY OF THEIR AUTHORIZED AGENTSOR REPRESENTATIVES

Freedom from unreasonable searches and seizures is declared a popular right Facts:
and for a search warrant to be valid, (a) it must be issued upon probable cause; Sometime in 1964, the petitioner and Reparations Commission entered into a conditional
(b) theprobable cause must be determined by the judge himself and not by the contract, subject to the condition that the title to and ownership of the vessel shall remain with
applicantor any other person; (c) in the determination of probable cause, the the Commission until full payment. Later on, petitioner entered into a contract with one Tomas
Velasco, authorizing the latter to supervise and manage the M/V JOLO LEMA. The vessel
judge mustexamine, under oath or affirmation, the complainant and such
however was however apprehended, searched and then seized by the Collector of Customs.
witnesses as theapplicant may produce; and (d) the warrant issued must A Seizure Identification proceeding was instituted against said vessel for smuggling into the
particularly describe theplace to be searched and persons or things to be seized. Philippines 1,408 sacks of Indonesian copra and 86 sacks of Indonesian coffee beans, in
In the case at bar, theexistence of probable cause was determined not be the violation of Section 2530 (a) and (k) of the Tariff and Customs Code of the Philippines. This
judge himself but by theapplicant.2. appeal taken by Lopez directly to the Supreme Court, upon the ground that only question sof
law would be taken up therein.

Issue:
The constitutional immunity against unreasonable searches and seizures is a Whether or not the Court of First Instance of Manila has jurisdiction to interfere with the
personal right which may be waived. The waiver may be either express or Seizure Identification proceeding No. 25/66 pending before the Commissioner of Customs,
implied. It is well-settled that to constitute a waiver of constitutional right, it on account of the Indonesian agricultural products smuggles into the Philippines through the
must appear that: (a) right exists, (b) persons involved had knowledge, either use of M/VJOLO LEMA
Page 4 of 33

HELD: The constitutional immunity from unreasonable searches and seizures, being
 Lopez maintains that whatever powers the Commissioner of Customs had, prior personal one, cannot be waived by anyone except 1) the person whose rights
thereto, over seizure identification proceedings had been transferred to the Philippine are invaded or 2) one who is expressly authorized to do so in his or her behalf.
Fisheries Commission. The Supreme Court said that this pretense is manifestly
devoid of merit. Said section 5 of Republic Act No. 3512 merely transfers to the Facts:
Philippine Fisheries Commission the powers, functions and duties of the Bureau of
Customs, the Philippine Navy and the Philippine Constabulary over fishing vessels  The group of Lt. Quijardo were sent to verify the presence of
and fishery matters. Such transfer should be construed in the light of section 1 of said CPP/NPA members in Dagupan City.
Republic Act No. 3512, reading. It is clear that the powers transferred to the  They put under surveillance the rented apartment of Rosemarie, sister
Philippine Fisheries Commission by Republic Act No. 3512 are limited to those of someone whom they earlier arrested.
relating to the "development, improvement, management and conservation of our  They interviewed Luzviminda Morados, a visitor of Rosemarie, who
fishery resources." All other matters, such as those concerning smuggling, stated that she worked with Bernie Mendoza alias Basilio Damaso, the
particularly of agricultural products, into the Philippines, are absolutely foreign to the appellant.
object and purpose of said Act and could not have been and were not transferred to  Together with Morados, they reached the house of Damaso where they
the aforementioned Commission. Seizure Identification proceeding No. 25/66 for saw Luz Tanciangco, a helper. Tanciangco then allowed the group to
the smuggling of Indonesian agricultural products into the Philippines is certainly enter inside the house.
beyond the jurisdiction of the Philippine Fisheries Commission.  The group of Lt. Quijardo entered the dwelling of Damaso without a
 The M/V JOLO LEMA is not subject to forfeiture, inasmuch as Davao is a port of valid warrant when the latter was absent. They requested the persons
entry. This is neither the time nor the place to pass upon the merits of this contention. in the house to allow them to look around. In one of the rooms, they
Suffice it to say that, if petitioner feels it is a good defense, the proper place to set it saw subversive materials which they confiscated. They likewise
up is in Seizure Identification proceeding No. 25/66. If the Commissioner of brought the persons found in the house to the headquarters for
Customs overrules such defense and decrees the forfeiture of the vessel, Lopez may investigation and the persons revealed that Damaso was the lessee of
appeal to the Court of Tax Appeals, whose decision may, in turn, be reviewed by the the house and owned the items confiscated.
Supreme Court. Lastly, petitioner argues that the Reparations Commission may not  Based on this, Damaso was charged with illegal possession of
unilaterally rescind its conditional contract of purchase and sale in his favor and that firearms.
the Commission must first seek a judicial declaration of rescission of said
contract. Well-settled is, however, the rule that a judicial action for the Whether the evidence is admissible? NO.
rescission of a contract is not necessary where the contract provides that it may be
revoked and cancelled for violation of any of its terms and conditions Held:

 The Court ruled that the law enforcers failed to comply with the
PEOPLE VS DAMASO requirements of a valid search and seizure. None of these exceptions
G.R. No. 93516, 212 SCRA 547, August 12, 1992 for a warrantless search is present in this case.
 Moreover, the constitutional immunity from unreasonable searches
"Maid allowed entry into the house" and seizures, being personal one, cannot be waived by anyone except
1) the person whose rights are invaded or 2) one who is expressly
The right against unreasonable searches and seizures is a personal right. authorized to do so in his or her behalf.
Page 5 of 33

 In this case, the records show that Damaso was not in his house at that definition and requirements of a valid stop-and-frisk as stated in People v. Chua -
time Luz, his alleged helper, allowed the authorities to enter. There that he should properly introduce himself and make initial inquiries, approach and
was no evidence that would establish the fact that Luz was indeed restrain a person who manifests unusual and suspicious conduct, in order to check
Damaso’s helper or if it was true that she was his helper, that Damaso the latter’s outer clothing for possibly concealed weapons.
had given her authority to open his house in his absence.
 Being a helper, she does not qualify as a person authorized to waive
such right in representation of her employer. ROMEO POSADAS y ZAMORA,
 Thus, the search being invalid for lack of warrant, the evidence vs.
THE HONORABLE COURT OF APPEALS and THE PEOPLE OF THE
obtained thereafter is inadmissible.
PHILIPPINES,
G.R. No. 89139 August 2, 1990
SUSAN ESQUILLO VS PEOPLE
Facts:
Gr no. 182010 August 25, 2010
While Pat. Ungab and Umpar were conducting a surveillance along Magallanes
Street,Davao City, they spotted petitioner carrying a "buri" bag and they noticed
FACTS:
him to beacting suspiciously.They approached the petitioner and identified
themselves asmembers of the INP. Petitioner attempted to flee but his attempt to
Susan Esquillo was convicted of the violation of the Dangerous Drugs Acts. On
get away wasthwarted by the two notwithstanding his resistance.They then
the time of the arrest, two police officers came to Esquillo and another person
checked the "buri" bag of the petitioner where they found one (1)
while they were transacting. While the officers were coming, one of the officers
caliber .38revolver, two (2) rounds of live ammunition for a .38 caliber gun, a
saw Esquillo hide a transparent plastic bag with white substance in it. When asked,
smoke (tear gas)grenade,
she fled but was eventually caught.
and two (2) live ammunitions for a .22 caliber gun.
the petitioner was askedto show the necessary license or authority to possess the
Esquillo argues that the arrest was invalid and that the officers planted evidence
firearms and ammunitions butfailed to do so.
against her.
Issue: Whether or not the warantless arrest and search was valid.
The lower cause said that the officers had probable cause to search Esquillo under
the stop-and-frisk doctrine.
Ruling:
An arrest without a warrant may be effected by a peace officer or private person,
ISSUE: Whether the arrest was valid.
amongothers, when in his presence the person to be arrested has committed, is
actuallycommitting, or is attempting to commit an offense; or when an offense has
RULING:
in fact justbeen committed, and he has personal knowledge of the facts indicating
that the personarrested has committed it.Contrary to the argument of the Solicitor
The SC denied the appeal.
General that when the two policemenapproached the petitioner, he was actually
committing or had just committed the offenseof illegal possession of firearms and
Firstly, the issue whether the arrest was valid was waived by the petitioner when
ammunitions in the presence of the police officersand consequently the search and
she did not quash it before arraignment. The issue was only raised the first time
seizure of the contraband was incidental to the lawfularrest in accordance with
during appeal on the appellate court.
Section 12, Rule 126 of the 1985 Rules on CriminalProcedure; At the time the
peace officers in this case identified themselves andapprehended the petitioner as
On regards her arrest, when the officer saw the white substance from a distance,
he attempted to flee they did not know that he hadcommitted, or was actually
the plain view doctrine was imposed. When searched the officers followed the
committing the offense of illegal possession of firearms andammunitions. They
Page 6 of 33

just suspected that he was hiding something in the buri bag. Theydid not know
what its contents were. The said circumstances did not justify an arrest without I. In Terry vs Ohio, a stop-and-frisk was defined as the vernacular designation of
warrant. the right of a police officer to stop a citizenon the street, interrogate him and pat
him for weapons: Where a police officer observes an unusual conduct which leads
him reasonably to conclude in light of his experience that criminal activity may be
a foot and that the persons with whom heis dealing may be armed and presently
Manalili vs CA dangerous, where in the course of investigating this behavior he identified himself
asa policeman and makes reasonable inquiries, and where nothing in the initial
Facts: stages of the encounter serves to dispel hisreasonable fear for his own or others'
This is a petition for certiorari seeking the reversal of CA’s decision in affirming safety, he is entitled for the protection of himself and others in the area to conduct
TC’s decision on convicting Manalili of illegal possession of prohibited drug a carefully limited search of the outer clothing of such persons in an attempt to
violating RA 6425.Police operatives Espiritu, Lumabas and driver Enriquez discover weapons which might be used toassault him. Such a search is a reasonable
conducted surveillance along the front of Kalookan Cemetery based on the search under the Fourth Amendment, and any weapon seized may properly be
information that drug addicts were roaming around in the area, saw a man who introduced in evidence against the person from whom they were taken. It did not,
appeared to be high on drugs and introduced themselves as policemen. Said man however abandon the rule that the police must, whenever practicable, obtain
avoided them and tried to resist, when they asked what the man was holding in his advance judicial approval of searches and seizures through the warrant procedure,
hand, the man held out his wallet and allowed Espiritu to examine it, who found excused only by exigent circumstances. As People vs Lacerna enumerated 5
what he suspected to be crushed mj leaves. The man was brought to the Anti- recognized exceptions to the rule against warrantless searches and seizures: 1)
Narcotics Unit and turned out to be Manalili. The substance found on search incidental to lawful arrest; 2) search of moving vehicles; 3) seizure in plain
Manalili’s wallet was sent to NBI Foresic Chemistry Section and was confirmed view; 4) customs search; 5) waiver of the accused of his rights against
as mj.Manalili’s version of the story was that early afternoon he was riding in a unreasonable searches and seizures. From Espiritu’s experience as a member of
tricycle when 3 policemen stopped the tricycle the Anti-Narcotics Unit of Caloocan City Police, Manalili’s suspicious behavior
and informed them of the suspected possession of mj, the policemen bodily was characteristic of drug addicts who were high.
searched both Manalili and the driver andupon finding nothing illegal on their
persons, let the driver go but brought Manalili along to the police station. Manalili II. SG’s contention that Manalili effectively waived the inadmissibility of the
while on the way to the station saw a neighbor whom he signaled to follow them evidence illegally obtained when he failed to raise this issue or object during trial.
and when he was again searched in the station, he was asked to strip his pants A valid waiver of right against unreasonable searches and seizures require the
where they found nothing illegal. Said neighbor then asked the policemen to let concurrence of these requisites: 1) the right to be waived existed; 2) the person
Manalili go seeing as they had not found anything illegal but Manalili was put on waiving it had knowledge; and 3) he/she had actual intention to relinquish the
a cell who was brought to a fiscal later that day and was told not to say anything right. In this case however, it is deemed that Manalili has waived such right for
despite his saying that the policemen had not found mj on his person. Said tricycle failure to raise its violation before the trial court, at the earliest opportunity
driver and neighbor testified on court as to possible. Issues not raised below cannot be pleaded for the first time on appeal.
how the 2 searches yielded nothing illegal on Manalili’s person.

Issues: III. Manalili’s contention that the charge was trumped up to extort money and
1. W/N evidence seized during a stop-and-frisk is admissible. testimonies of the arresting officers were inconsistent, it held that the trial court’
2. W/N Manalili’s actions constituted a waiver of his rights. s assessment of the credibility of the witnesses particularly when affirmed by CA
3. W/N the evidence is sufficient to prove Manalili’s guilt. is
accorded great weight and respect as it had opportunity to observe their demeanor
Ruling: and deportment as they testified before it.The elements of illegal possession of mj
Page 7 of 33

are: allegedly the previous Saturday, 25 August 1990, likewise at Plaza Miranda, Yu
a) the accused is in possession of an item or object which is identified to be a saw Malacat and 2 others attempt to detonate a grenade). Upon searching Malacat,
prohibited drug; b) such possession is not authorized by law; and c) the accused Yu found a fragmentation grenade tucked inside the latter’s “front waist line.”
freely and consciously possessed the said drug. The substance found on Manalili’ Yu’s companion, police officer Rogelio Malibiran, apprehended Abdul Casan
s wallet was identified as mj which was prohibited and knowingly without from whom a .38 caliber revolver was recovered. Malacat and Casan were then
authority.Considering that he was high and tried to avoid and resist, such behavior brought to Police Station 3 where Yu placed an “X” mark at the bottom of the
clearly shows that he knew he was holding mj andit was prohibited by law. grenade and thereafter gave it to his commander. Yu did not issue any receipt for
the grenade he allegedly recovered from Malacat. On 30 August 1990, Malacat
was charged with violating Section 3 of Presidential Decree 1866. At arraignment
on 9 October 1990, petitioner, assisted by counsel de officio, entered a plea of not
Malacat v CA guilty. Malacat denied the charges and explained that he only recently arrived in
283 SCRA 159 (December 12, 1997) Manila. However, several other police officers mauled him, hitting him with
benches and guns. Petitioner was once again searched, but nothing was found on
Facts: him. He saw the grenade only in court when it was presented. In its decision dated
On 27 August 1990, at about 6:30 p.m., allegedly in response to bomb threats 10 February 1994 but promulgated on 15 February 1994, the trial court ruled that
reported seven days earlier, Rodolfo Yu of the Western Police District, the warrantless search and seizure of Malacat was akin to a
Metropolitan Police Force of the Integrated National Police, Police Station No. 3,
Quiapo, Manila, was on foot patrol with three other police officers (all of them in Issue: Whether the search made on Malacat is valid, pursuant to the exception of
uniform) along Quezon Boulevard, Quiapo, Manila, near the Mercury Drug store “stop and frisk.”
at Plaza Miranda. They chanced upon two groups of Muslim-looking men, with
each group, comprised of three to four men, posted at opposite sides of the corner Held: The general rule as regards arrests, searches and seizures is that a warrant
of “stop and frisk,” where a “warrant and seizure can be effected without is needed in order to validly effect the same. The Constitutional prohibition against
necessarily being preceded by an arrest” and “whose object is either to maintain unreasonable arrests, searches and seizures refers to those effected without a
the status quo momentarily while the police officer seeks to obtain more validly issued warrant, subject to certain exceptions. As regards valid warrantless
information”; and that the seizure of the grenade from Malacat was incidental to arrests, these are found in Section 5, Rule 113 of the Rules of Court. A warrantless
a lawful arrest. The trial court thus found Malacat guilty of the crime of illegal arrest under the circumstances contemplated under Section 5(a) has been
possession of explosives under Section 3 of PD 1866, and sentenced him to suffer denominated as one “in flagrante delicto,” while that under Section 5(b) has
the penalty of not less than 17 years, 4 months and 1 day of Reclusion Temporal, been described as a “hot pursuit” arrest. Turning to valid warrantless searches,
as minimum, and not more than 30 years of Reclusion Perpetua, as maximum. On they are limited to the following: (1) customs searches; (2)search of moving
18 February 1994, Malacat filed a notice of appeal indicating that he was appealing vehicles; (3) seizure of evidence in plain view; (4) consent searches; (5) a search
to the Supreme Court. However, the record of the case was forwarded to the Court incidental to alawful arrest; and (6) a “stop and frisk.”
of Appeals (CA-GR CR 15988). In its decision of 24 January 1996, the Court of
Appeals affirmed the trial court. Manalili filed a petition for review with the
The concepts of a “stop-and-frisk” and of a search incidental to a lawful arrest
Supreme Court.
must not be confused. These two types of warrantless searches differ in terms of
Quezon Boulevard -near the Mercury Drug Store. These men were acting
the requisite quantum of proof before they may be validly effected and in their
suspiciously with “their eyes moving very fast.” Yu and his companions allowable scope. In a search incidental to a lawful arrest, as the precedent arrest
positioned themselves at strategic points and observed both groups for about 30 determines the validity of the incidental search. Here, there could have been no
minutes. The police officers then approached one group of men, who then fled in valid in flagrante delicto or hot pursuit arrest preceding the search in light of the
different directions. As the policemen gave chase, Yu caught up with and lack of personal knowledge on the part of Yu, the arresting officer, or an overt
apprehended Sammy Malacat y Mandar (who Yu recognized, inasmuch as physical act, on the part of Malacat, indicating that a crime had just been
Page 8 of 33

committed, was being committed or was going to be committed. Plainly, the search They alleged that there is no specific target house to be search and that there is no
conducted on Malacat could not have been one incidental to a lawful arrest. On search warrant or warrant of arrest served. Most of the policemen are in their
the other hand, while probable cause is not required to conduct a “stop and civilian clothes and without nameplates or identification cards. The residents were
frisk,” it nevertheless holds that mere suspicion or a hunch will not validate a rudely rouse from their sleep by banging on the walls and windows of their houses.
“stop and frisk.” A genuine reason must exist, in light of the police officer’s The residents were at the point of high-powered guns and herded like cows. Men
experience and were ordered to strip down to their briefs for the police to examine their tattoo
surrounding conditions, to warrant the belief that the person detained has weapons marks. The residents complained that they're homes were ransacked, tossing their
belongings and destroying their valuables. Some of their money and valuables had
concealed about him. Finally, a “stop -and- frisk” serves a two-fold interest: (1)
disappeared after the operation. The residents also reported incidents of maulings,
the general interest of effective crime prevention and detection, which underlies
spot-beatings and maltreatment. Those who were detained also suffered mental
the recognition that a police officer may, under appropriate circumstances and in
and physical torture to extract confessions and tactical informations. The
an appropriate manner, approach a person for purposes of investigating possible
respondents said that such accusations were all lies. Respondents contends that the
criminal behavior even without probable cause; and (2) the more pressing interest
Constitution grants to government the power to seek and cripple subversive
of safety and self-preservation which permit the police officer to take steps to
movements for the maintenance of peace in the state. The aerial target zoning were
assure himself that the person with whom he deals is not armed with a deadly
intended to flush out subversives and criminal elements coddled by the
weapon that could unexpectedly and fatally be used against the police officer.
communities were the said drives were conducted. They said that they have
Here, there are at least three (3) reasons why the “stop-and-frisk” was invalid: intelligently and carefully planned months ahead for the actual operation and that
First, there is grave doubts as to Yu’s claim that Malacat was a member of the local and foreign media joined the operation to witness and record such event.
group which attempted to bomb Plaza Miranda 2 days earlier. This claim is neither
supported by any police report or record nor corroborated by any other police
officer who allegedly chased that group. Second, there was nothing in Malaca Issue: Whether or Not the saturation drive committed consisted of violation of
t’s behavior or conduct which could have reasonably elicited even mere human rights.
suspicion other than that his eyes were “moving very fast” — an observation
which leaves us incredulous since Yu and his teammates were nowhere near
Malacat and it was already 6:30 p.m., thus presumably dusk. Malacat and his Held: It is not the police action per se which should be prohibited rather it is the
companions were merely standing at the corner and were not creating any procedure used or the methods which "offend even hardened sensibilities" .Based
commotion or trouble. Third, there was at all no ground, probable or otherwise, to on the facts stated by the parties, it appears to have been no impediment to securing
believe that Malacat was armed with a deadly weapon. None was visible to Yu, for search warrants or warrants of arrest before any houses were searched or
as he admitted, the alleged grenade was “discovered” “inside the front individuals roused from sleep were arrested. There is no showing that the
waistline” of Malacat, and from all indications as to the distance between Yu and objectives sought to be attained by the "aerial zoning" could not be achieved even
Malacat, any telltale bulge, assuming that Malacat was indeed hiding a grenade, as th rights of the squatters and low income families are fully protected. However,
could not have been visible to Yu. What is unequivocal then are blatant violations the remedy should not be brought by a tazpaer suit where not one victim
of Malacat’s rights solemnly guaranteed in Sections 2 complaints and not one violator is properly charged. In the circumstances of this
and 12(1) of Article III of the Constitution. taxpayers' suit, there is no erring soldier or policeman whom the court can order
prosecuted. In the absence of clear facts no permanent relief can be given.
In the meantime where there is showing that some abuses were committed, the
GUANZON VS. DE VILLA [181 SCRA 623; G.R. 80508; 30 JAN 1990] court temporary restraint the alleged violations which are shocking to the senses.
Petition is remanded to the RTC of Manila.
Facts: The 41 petitioners alleged that the "saturation drive" or "aerial target
zoning" that were conducted in their place (Tondo Manila) were unconstitutional.
Page 9 of 33

UMIL VS. RAMOS [187 SCRA 311; G.R. NO. 81567; 3 OCT 1991]

FACTS
Facts: On 1 February 1988, military agents were dispatched to the St. Agnes
Hospital, Roosevelt Avenue, Quezon City, to verify a confidential information - Municipal Judge Samulde conducted a preliminary
which was received by their office, about a "sparrow man" (NPA member) investigation upon a complaint for robbery. After
who had been admitted to the said hospital with a gunshot wound. That the
wounded man in the said hospital was among the five (5) male "sparrows" who making a preliminary investigation based on the
murdered two (2) Capcom mobile patrols the day before, or on 31 January
1988 at about 12:00 o'clock noon, before a road hump along Macanining St., affidavits of the complainant and her witnesses and
Bagong Barrio, Caloocan City. The wounded man's name was listed by the
counter-affidavits of the respondent and his
hospital management as "Ronnie Javellon," twenty-two (22) years old of
Block 10, Lot 4, South City Homes, Biñan, Laguna however it was disclosed witnesses, Judge Samulde transmitted the records of
later that the true name of the wounded man was Rolando Dural. In view of
this verification, Rolando Dural was transferred to the Regional Medical the case to Provincial Fiscal Salvani with his finding
Servicesof the CAPCOM, for security reasons. While confined thereat, he was
that "there is prima facie evidence of robbery as
positively identified by the eyewitnesses as the one who murdered the 2
CAPCOM mobile patrols. charge in the complaint". The fiscal returned the

records on the ground that Judge Samulde failed to


Issue:WON Rolando was lawfully arrested
include the warrant of arrest against the accused as

provided in Sec 5, Rule 112 of the 1985 Rules on


Held: Rolando Dural was arrested for being a member of the NPA, an
outlawed subversive organization. Subversion being a continuing offense, the Criminal Procedure. Judge Samulde sent back the
arrest without warrant is justified as it can be said that he was committing as
records to Fiscal Salvani. He pointed out that under
offense when arrested. The crimes rebellion, subversion, conspiracy or
proposal to commit such crimes, and crimes or offenses committed in Sec 6, Rule 112, he may issue a warrant of arrest if
furtherance therefore in connection therewith constitute direct assaults against
the state and are in the nature of continuing crimes. he is satisfied "that a probable cause exists and that

there is a necessity of placing the respondent under

SAMULDE v SALVANI immediate custody in order not to frustrate the ends


Page 10 of 33

of justice, " implying that, although he found that a HELD

probable cause existed, he did not believe that the NO

accused should be immediately placed under Ratio 3 conditions must concur for the issuance of

custody. Hence, he refused to issue a warrant of the warrant of arrest. The investigating judge must:

arrest. (a) have examined in writing and under oath the

- A special civil action of mandamus was filed in the complainant and his witnesses by searching

RTC by Provincial Fiscal Salvani against Judge questions and answers;

Samulde to compel the latter to issue a warrant of (b) be satisfied that a probable cause exists; and

arrest. The RTC dismissed the petition but (c) that there is a need to place the respondent

nevertheless ordered Judge Samulde to issue a under immediate custody in order not to frustrate the

warrant of arrest, and to transmit the warrant to the ends of justice.

Provincial Fiscal for appropriate action. He further Reasoning The mandatory provision that the

advised the Municipal Judge "that henceforth he investigating judge "must issue a warrant of arrest" if

adheres to the same rule in similar cases where he he finds probable cause that the respondent

conducts a preliminary investigation with a finding of committed the crime charged, found in all previous

a prima facie or probable cause." Unconvinced, Judge rules of criminal procedure, from General Orders No.

Samulde appealed to this Court. 58 down to Rule 112 of the 1964 Revised Rules of

ISSUE Court, is absent in Section 1 of the 1985 Rules on

WON a judge may be compelled to issue a warrant of Criminal Procedure. It is not obligatory, but merely

arrest upon a finding of probable cause discretionary, upon the investigating judge to issue a
Page 11 of 33

warrant for the arrest of the accused, for the – A petition for certiorari has been filed to invalidate the order of Judge
Casanova which quashed search warrant issued by Judge Bacalla and declared
determination of whether a probable cause exists inadmissible for any purpose the items seized under the warrant.
and whether it is necessary to arrest the accused in – An application for a search warrant was made by S/Insp Brillantes against
order not to frustrate the ends of justice, is left to his Mr. Azfar Hussain who had allegedly in his possession firearms and explosives
at Abigail Variety Store, Apt 1207 Area F. Bagon Buhay Avenue, Sarang
sound judgment or discretion. In this particular case, Palay, San Jose Del Monte, Bulacan.

since the robbery charge was the offshoot of a – The following day Search Warrant No. 1068 was issued but was served not
at Abigail Variety Store but at Apt. No. 1, immediately adjacent to Abigail
boundary dispute between two property owners, the Variety Store resulting in the arrest of 4 Pakistani nationals and the seizure of
a number of different explosives and firearms.
investigating judge did not believe there was any

danger of the accused absconding before the filing of


ISSUE:

1. WON a search warrant was validly issued as regard the apartment in


PEOPLE OF THE PHILIPPINES, represented by Provincial Prosecutor
which private respondents were then actually residing, or more
FAUSTINO T.
explicitly, WON that particular apartment had been specifically
CHIONG, petitioner, described in the warrant.

COURT OF APPEALS, JUDGE CAESAR CASANOVA, Presiding


Judge, Regional Trial
HELD:
Court, Branch 80, Malolos, Bulacan, AZFAR HUSSAIN, MOHAMMAD
In applying for a search warrant, the police officers had in their mind the first
SAGED,
four (4) separate apartment units at the rear of ABIGAIL VARIETY STORE
MUJAHID KHAN, MOHAMMAD ASLAM, and MEHMOOD in Quezon City to be the subject of their search.
ALI, respondents.
The same was not, however, what the Judge who issued the warrant had in
mind, AND WAS NOT WHAT WAS ULTIMATELY DESCRIBED IN THE
SEARCH WARRANT.

FACTS:
Page 12 of 33

As such, any evidence obtained from the place searched which is different OSCAR L. GOZOS, Presiding Prosecutor of Batangas, EDNA DYOGI, et al.,
from that indicated in the search warrant is inadmissible in evidence for any petitioners, vs. HON. PATERNO C. TAC-AN, Presiding Judge, Regional
purpose and in any proceeding. Trial Court, Branch 84, Batangas City; SPO2 JAIME V. BLANCO, SPO3
PEDRO CASTILLO, SPO3 CIRIANO S. SULIT, SPO4 ANIANO
The ambiguity lies outside the instrument, arising from the absence of a ATIENZA, and SPO1 ILDEFONSO CASTILLO, respondents.
meeting of minds as to the place to be searched between the applicants for the
warrant and the Judge issuing the same; and what was done was to substitute These are two special civil actions for certiorari under Rule 65, 1 of the Rules
for the place that the Judge had written down in the warrant, the premises that of Court seeking to set aside orders, dated October 18, 1995, November 22,
the executing officers had in their mind. This should not have been done. 1995, and January 3, 1996, issued by respondent Judge Paterno Tac-an of the
Regional Trial Court, Batangas City. A summary of the gist of the orders
It is neither fair nor licit to allow police officers to search a place different from follows:
that stated in the warrant on the claim that the place actually searched —
although not that specified in the warrant — is exactly what they had in view 1.) Order dated October 18, 1995 Respondent judge directed the Office
when they applied for the warrant and had demarcated in their supporting of the Provincial Prosecutor, Batangas City to amend the information filed on
evidence. What is material in determining the validity of a search is the place June 26, 1995 for the death of Gilbert Dyogi by the Office of the Deputy
stated in the warrant itself, not what the applicants had in their thoughts, or had Ombudsman for the Military against private respondents SPO2 Jaime Blanco,
represented in the proofs they submitted to the court issuing the warrant. SPO3 Pedro Castillo, SPO3 Ciriaco Sulit, SPO2 Aniano Atienza, and SPO1
Ildefonso Castillo, members of the Philippine National Police, San Jose,
The place to be searched, as set out in the warrant, cannot be amplified or Batangas. The amendment would reduce the charge against private respondent
modified by the officers’ own personal knowledge of the premises, or the SPO2 Jaime Blanco from murder to homicide and drop the rest of the private
evidence they adduced in support of their application for the warrant. Such a respondents from the information.
change is proscribed by the Constitution which requires inter alia the search
warrant to particularly describe the place to be searched as well as the persons 2.) Order dated November 22, 1995 Acting on the motion for
or things to be seized. reconsideration filed on November 7, 1995 by the complainant Edna Dyogi,
respondent judge directed the Office of the Provincial Prosecutor, Batangas
It would concede to police officers the power of choosing the place to be City to amend the information by including private respondent Pedro Castillo
searched, even if it not be that delineated in the warrant. It would open wide as an accomplice in the crime. However, respondent judge denied the motion
the door to abuse of the search process, and grant to officers executing a search insofar as it sought to include the other private respondent as accused in the
warrant that discretion which the Constitution has precisely removed from criminal case.
them. The particularization of the description of the place to be searched may
properly be done only by the Judge, and only in the warrant itself; it cannot be 3.) Order dated January 3, 1996 Respondent judge Tac-an denied the
left to the discretion of the police officers conducting the search. second motion for reconsideration filed by complainant Edna Dyogi,
questioning the authority of respondent to require the Provincial Prosecutor to
[G.R. No. 123191. December 17, 1998] amend the information.
Page 13 of 33

The petition in G.R. No. 123442 was filed by the Solicitor General on behalf Castillo, Sulit, Atienza, and Ildefonso Castillo with murder in an information
of the People of the Philippines, while the petition in G.R. No. 123191 was filed with the Regional Trial Court of Batangas City. The information alleged:
filed by complainant Edna Dyogi. In a manifestation, dated February 7, 1996,
complainant adopted the petition of the Solicitor General, for which reason the That sometime on February 9, 1995 at Poblacion, San Jose, Batangas, and
Court directed in its resolution, dated August 5, 1996, the consolidation of the within the jurisdiction of this Honorable Court, accused SPO2 Jaime Blanco,
two cases. SPO3 Pedro V. Castillo, SPO3 Ciriaco R. Sulit, SPO2 Aniano Atienza and
SPO1 Ildefonso R. Castillo, all public officers, being then members of the
The facts are as follows: Philippine National Police of San Jose, Batangas, conspiring and
confederating, and taking advantage of their official positions, with treachery
It appears that on the night of February 9, 1995, the officials, teachers, and and taking advantage of their superior strength, with intent to kill, did then and
students of the Concepcion Aguila Memorial College in San Jose, Batangas there, wilfully, unlawfully and feloniously shoot with a firearm one Gilbert
organized a school party. While the party was going on, the principal, Dyogi, who was then weak and defenseless, thereby inflicting upon the latter,
Felizardo Aguila, was informed that several men, who appeared to be drunk, a fatal wound which caused his instantaneous death.v[5]
were trying to force their way through the main gate. One of the men seemed
armed with a handgun. After calling the police, Aguila went to the main gate, Private respondents Blanco, Pedro Castillo, Sulit, Atienza, and Ildefonso
where he asked the men what their business was.i[1] Castillo filed a Motion to Hold Issuance of Warrant and Motion to Quash with
Motion to Set Incidents for Hearing in which they prayed:
At this point, private respondents Blanco and Atienza arrived at the school.
They were shortly joined by private respondents Pedro Castillo, Sulit, and WHEREFORE, it is respectfully prayed that the Honorable Court first
Ildefonso Castillo, who were all members of the Philippine National Police of determine whether or not there is probable cause against all the accused before
San Jose, Batangas. They demanded from the man armed with a handgun, who the issuance of a warrant of arrest, and the case be set for open hearing with
later turned out to be the victim Gilbert Dyogi, that he surrender his gun and notice to all the parties for that purpose and that in the meantime, the issuance
go with them to the station.ii[2] Gilbert Dyogi gave the handgun to the of a warrant of arrest be held in abeyance, in the interest of justice, and
respondents, who then asked to see his license. He produced a sheet of paper thereafter the information be quashed and/or dismissed as against the accused
from his wallet which he handed to private respondents. After allowing them against whom there is no probable cause.vi[6]
to inspect the weapon and the alleged license, Gilbert Dyogi asked the private
respondents to give them back to him. However, private respondent Blanco, Petitioner Edna Dyogi opposed the motion. She maintained that there was
who had the gun, refused to do so.iii[3] probable cause for the filing of the case against private respondents Blanco,
Pedro Castillo, Sulit, Atienza, and Ildefonso Castillo. Respondent Tac-an
What exactly followed is unclear. Before long the two were grappling for heard the parties on their motion after which, in an order dated October 8,
possession of the gun. Apparently, Blanco pulled out his sidearm and fired at 1995, he ruled as follows:
Gilbert Dyogi twice.iv[4]
WHEREFORE, it is the opinion of this Court that there is probable cause to
In connection with the fatal incident, Gerald Varez, Investigator in the Office hold SPO2 Jaime V. Blanco to stand trial for homicide only. His co-accused
of the Ombudsman for the Military, charged private respondents Blanco, Pedro SPO3 Pedro V. Castillo, SPO3 Ciriaco R. Sulit, SPO2 Aniano Atienza and
Page 14 of 33

Ildefonso R. Castillo are hereby discharged from the information for 1. That the finding of probable cause for homicide only against accused
insufficiency of evidence. Jaime V. Blanco for shooting to death Gilbert Dyogi on February 9, 1995 at
around eleven in the evening is hereby maintained.
.
2. That there is probable cause to hold accused PEDRO V. CASTILLO
Let a warrant of arrest be issued for the apprehension of SPO2 Jaime V. Blanco liable as accomplice only by cooperating with a simultaneous act of
who is reportedly assigned at the Provincial Command Headquarters, PNP, intervention which act was not indispensable, but facilitated the killing of
Puerto Princesa, Palawan. The amount of P50,000.00 in cash and/or property Dyogi by Blanco, by then holding Ben Flores and preventing him from
is hereby fixed for his provisional liberty. approaching and dissuading Dyogi from insisting in retaining his firearm in
. his person or recovering the possession thereof from said Blanco or in
preventing Flores from pacifying Blanco from shooting Dyogi. Such act is
The Provincial Prosecutor or his authorized assistants are hereby directed to contrary to law.
file an Amended Information for purposes of formality and convenience within
five (5) days from receipt thereof. Let a Warrant of Arrest be issued for his apprehension. Bail for his temporary
liberty is fixed at P10,000.00. If an insurance company should serve as
. bondsman, the authority of the issuing person must be supported by a board
resolution to that effect;
. This duty of the judge necessarily includes the determination of the proper
offense under which the accused is to undergo trial. To do otherwise would be 3 As regards the rest of the accused, namely: Ciriaco P. Sulit, Aniano
tantamount to dereliction of duty. Any pronouncement to the contrary has been Atienza, and Ildefonso R. Castillo, the finding that there is no probable cause
repealed by the 1987 Constitution and by the latest Supreme Court rulings on is hereby maintained. However, the prosecution may, at its option, within a
this matter.vii[7] period of twenty (20) days from receipt of this Order, adduce additional
evidence, if any, against them before this Court:
Petitioner Dyogi moved for a reconsideration, arguing that the crime
committed was murder, and not homicide, and that there was probable cause 4 That the information shall be amended accordingly by the public
to believe that private respondents Blanco, Pedro Castillo, Sulit, Atienza, and prosecutor, as a matter of form and for convenience in the proceedings. The
Ildefonso Castillo were guilty of the offense. In his order, dated November 22, public prosecutors office is hereby enjoined not only to show obedience to
1995, respondent judge partially reconsidered his order by directing the duty and fairness, but moral courage as well, to follow the path of legal
inclusion of respondent Pedro Castillo in the information, not as principal but rectitude, which hitherto was covered with mist but is now illuminated by the
only as an accomplice. In all other respect, he affirmed his previous order. The guidance of the Constitution unto this Court. The Court is now the final arbiter
dispositive portion of his order, dated November 22, 1996, reads: under the 1987 Constitution in determining probable cause before a Warrant
of Arrest shall issue whereas before it was the prerogative of the public
WHEREFORE, PREMISES CONSIDERED, this Court resolves: prosecutor to so determine and whose own ascertainment or exercise of
discretion may not be interfered with by the Court, save in cases of grave abuse
of discretion, and whose information filed may not be ordered to be amended
Page 15 of 33

by the Court. Any public officer must yield to the Constitution as he has the (a) Provincial or city fiscals and their assistants;
sworn duty to obey and defend it.viii[8]
(b) Judges of the Municipal Trial Courts and Municipal Circuit Trial
On December 8, 1995, petitioner Edna Dyogi filed another motion for Courts;
reconsideration, contending that the power to determine the nature of the
offense to be charged was vested in the Provincial Prosecutor and not in the (c) National and Regional state prosecutors; and
Regional Trial Court. However, petitioners motion was denied by the court in (d) Such other officers as may be authorized by law.
its order, dated January 3, 1996, on the ground that it was actually a second
motion for reconsideration which is not allowed to be filed. Hence, these Their authority to conduct preliminary investigation shall include all crimes
petitions for certiorari filed by the Solicitor General and by the complainant in cognizable by the proper court in their respective territorial jurisdictions.
the criminal case, Edna Dyogi.
Thus, as provided in Rule 112, 2(d), other officers may be authorized by law
Petitioners raise the following issues: to conduct preliminary investigations. Indeed, under R.A. No. 6770, otherwise
known as the Ombudsman Act of 1989, investigators of the Office of the
Whether or not the Respondent Judge acted without or in excess of jurisdiction
Ombudsman may conduct preliminary investigations of cases involving public
and/or with grave abuse of discretion amounting to lack of jurisdiction: officers. Thus, 15(1) of the said law provides:
1. in conducting another preliminary investigation in a proceeding to The Office of the Ombudsman shall have the following powers, functions and
determine probable cause for the issuance of warrants of arrest.
duties:
2. in ordering the amendment of the crime charged in the information (1) Investigate and prosecute on its own or on complaint by any person,
from murder to homicide and the number and nature of the participation of the any act or omission of any public officer or employee, office or agency, when
accused charged in the information filed by the Office of the Deputy such act or omission appears to be illegal, unjust, improper, or inefficient. It
Ombudsman for the Military.ix[9]
has primary jurisdiction over cases cognizable by the Sandiganbayan and, in
We find both petitions to be meritorious. the exercise of this primary jurisdiction, it may take over, at any stage, from
any investigatory agency of Government, the investigation of such cases.
Rule 112, 1 of the Rules of Court defines a preliminary investigation as an
inquiry or proceeding for the purpose of determining whether there is The investigators of the Office of the Ombudsman have concurrent jurisdiction
sufficient ground to engender a well-founded belief that a crime cognizable by with public prosecutors to conduct preliminary investigations in all cases
the Regional Trial Court has been committed and that the respondent is involving public officers, whether falling under the jurisdiction of the
probably guilty thereof, and should be held for trial. Rule 112, 2 of the Rules Sandiganbayan or the regular courts. For this purpose, Administrative Order
of Court enumerates the officers authorized to conduct preliminary No. 8, dated November 8, 1990, of the Office of the Ombudsman provides:
investigations, as follows: For purposes of investigation and prosecution, Ombudsman cases involving
The following may conduct a preliminary investigation: criminal offenses may be subdivided into two classes, to wit: (1) those
Page 16 of 33

cognizable by the Sandiganbayan, and (2) those falling under the jurisdiction respondent judge did not only determine the existence of probable cause for
of the regular courts. the issuance of warrants of arrest, but also what the charge should be and who
should be charged. In so doing, respondent judge exceeded his authority.
The power to investigate or conduct a preliminary investigation in any
Ombudsman case may be exercised by an investigator or prosecutor of the To justify his orders, respondent judge invokes Art. III, 2 of the 1997
Office of the Ombudsman, or by any Provincial or City Prosecutor or their Constitution, which provides that no search warrant or warrant of arrest shall
assistants, either in their regular capacities or as deputized Ombudsman issue except upon probable cause to be determined personally by the judge
Prosecutors. after examination under oath or affirmation of the complainant and the
witnesses he may produce, and particularly describing the place to be searched
Thus, while the power of an investigator of the Office of the Ombudsman for and the persons or things to be seized. However, this provision applies to the
the Military is undoubted, no similar authority is vested in judges of Regional issuance of arrest and search warrants, which should be distinguished from a
Trial Courts as they are not among those mentioned in Rule 112, 2 as preliminary investigation. As already stated, the determination of probable
authorized to conduct preliminary investigations. As this Court pointed out in cause for the issuance of such orders is vested in the courts, but the conduct of
Castillo v. Villaluz:x[10] preliminary investigations is entrusted to the executive branch, with the
Judges of Regional Trial Courts (formerly Courts of First Instance) no longer exception of inferior court judges. According to this Court in People v.
Inting:xii[12]
have authority to conduct preliminary investigations. That authority, at one
time reposed in them under Sections 13, 14 and 16, Rule 112 of the Rules of Judges and prosecutors alike should distinguish the preliminary inquiry which
Court of 1964, was removed from them by the 1985 Rules on Criminal determines probable cause for the issuance of a warrant of arrest from the
Procedure, effective on January 1, 1985, which deleted all provisions granting preliminary investigation proper which ascertains whether the offender should
that power to said judges. be held for trial or release. The determination of probable cause for the warrant
The 1988 Amendments to the 1985 Rules on Criminal Procedure, declared of arrest is made by the Judge. The preliminary investigation proper whether
effective on October 1, 1988, did not restore that authority to Judges of or not there is reasonable ground to believe that the accused is guilty of the
Regional Trial Courts; said amendments did not in fact deal at all with the offense charged and, therefore, whether or not he should be subjected to the
officers or courts having authority to conduct preliminary investigations. expenses, rigors and embarrassment of trial is the function of the prosecutor.

As explained in Salta v. Court of Appeals,xi[11] the preliminary investigation .


proper is, therefore, not a judicial function. It is a part of the prosecutions job, We reiterate that preliminary investigation should be distinguished as to
a function of the executive. whether it is an investigation for the determination of a sufficient ground for
It is evident that in this case, respondent judge conducted an inquiry, not only the filing of the information or it is an investigation for the determination of a
probable cause for the issuance of the warrant of arrest. The first kind of
for the purpose of determining whether there was probable cause to order the
arrest of private respondents Blanco, Pedro Castillo, Sulit, Atienza, and preliminary investigation is executive in nature. It is part of the prosecutions
Ildefonso Castillo, but for the purpose of determining whether there was job. The second kind of preliminary investigation which is more properly
sufficient evidence to prosecute them as well. Indeed, in his questioned orders,
Page 17 of 33

called preliminary examination is judicial in nature and is lodged with the WHEREFORE, the petition is hereby granted and the orders, dated October
judge. 18, 1995, November 22, 1995, and January 3, 1996, of respondent Judge
Paterno Tac-an are ANNULLED and SET ASIDE.
Private respondents Pedro Castillo, Sulit, Atienza, and Ildefonso Castillo
defend the assailed orders on the theory that, since in their Motion to Hold SO ORDERED
Issuance of Warrant and Motion to Quash with Motion to Set Incidents for
Hearing they also sought to quash the original information filed by
Ombudsman Investigator Varez of the Office of the Ombudsman for the
military, respondent judge properly acted in hearing the motion to quash.
However, private respondents sought to quash the information on the theory
that there was no probable cause against them. Lack of probable cause is not
one of the grounds for a motion to quash under Rule 117, 3 of the Rules of NEMESIO PRUDENTE vs Hon Judge ABELARDO M. DAYRIT
Criminal Procedure. A motion to quash should be based on a defect in the
information which is evident on its face. The guilt or innocence of the accused,
their degree of participation, and the mitigating, aggravating, or alternative FACTS:
circumstances which should be appreciated are properly the subject of trial on
the merits rather than a motion to quash. As held in Cruz v. People:xiii[13] The Chief of the Intelligence Special Action Division (ISAD) filed with the
Regional Trial Court (RTC) Manila, Judge Abelardo Dayrit, for the issuance
It must here be stressed that a preliminary investigation is merely inquisitorial, of Search Warrant for violation of PD No. 1866 (Illegal Possession of Firearm,
and it is often the only means of discovering the persons who may be etc). In the deposition of witness (P/Lt. Florencio C. Angeles), it was made
reasonable charged with a crime, to enable the prosecutor to prepare his mentioned of “result of our continuous surveillance conducted for several
complaint or information. It is not a trial of the case on the merits and has no days. We gathered information from verified sources that the holders of said
purpose except that of determining whether a crime has been committed and firearms and explosives as well as ammunitions aren’t licensed to possess said
whether there is probable cause to believe that the accused is guilty thereof; firearms and ammunition. Further, the premises is a school and the holders of
and it does not place the persons against whom it is taken in jeopardy. these firearms are not student who were not supposed to possess firearms,
explosives and ammunitions.
The established rule is that a preliminary investigation is not the occasion for
the full and exhaustive display of the parties evidence; it is for the presentation
of such evidence only as may engender a well-grounded belief that an offense
has been committed and that the accused is probably guilty thereof. Person to be searched in Nemesio Prudente at the Polytechnic University of
the Philippines, Sta. Mesa, Sampaloc, Manila, has in his control or possession
Hence, notwithstanding the contrary opinion of the judge regarding the firearms, explosives hand grenades and ammunitions which are illegally
designation of the offense committed, for as long as he finds probable cause possesses at the office of Department of Military Science and Tactics and at
for the offense charged, he should issue a warrant of arrest against the accused the office of the President.
for the crime charged in the information.
Page 18 of 33

affirmation of the complainant, and that witnesses he may produce and


particularly describing the place to be searched and the persons and things to
Petitioner moved to quash the Search Warrant. He claimed that: be seized. The probable cause must be in connection with one specific offense
1. Petitioners, had no personal knowledge of the facts and the Judge must, before issuing Search Warrant, personally examine in the
form of searching questions and answers, In writing and under oath, the
2. The examination of the said witness was not in form of searching complainant and any witnesses he may produce, on facts personally known to
questions and answers them and attach to the record their sworn statements together with any
affidavits submitted.
3. Search warrant was a general warrant

4. Violation of Circular No. 19 of the Supreme Court in that the complainant


failed to allege under oath that the issuance of the search warrant on a Saturday, “Probable Cause” for a valid search warrant, has been defined as such facts
urgent. and circumstances which would lead a reasonably discreet and prudent man
to believe that an offense has been committed, and that objects sought in
connection which the offense are in the place sought to be searched.
ISSUE: - This probable case must be shown to be personal knowledge and of the
complainant and witnesses he may produce and not based on mere hearsay.
Whether or not the search and seizure was valid?

PARTICULARITY
HELD:
For violation of PD 1866 (Illegal Possession of Firearms, etc.) while the said
Search Warrant annulled and set aside.
decree punishes several offenses, the alleged violation in this case was,
qualified by the phrase illegal possession of firearms etc. - - Reformed to
ammunitions and explosives. In other words, the search warrant was issued for
RATIONALE: the specific offense of illegal possession of firearms and explosives. Hence,
the failure of the Search Warrant to mention the particular provision of PD1-
Valid search warrant to issue, there must be probable cause, which is to be
866 that was violated is not of such gravity as to call for the invalidation of
determined personally by the Judge, after examination under oath and
this case.

GR 121234 23 August 1995

Webb vs De Leon Facts: Hubert Webb was one of the accused in the high-profile case Vizconde
massacre. Preliminary investigation was provided by NBI and the case was
Page 19 of 33

disqualification of any member of the DOJ Panel on the ground of bias


raffled to Judge Zosimo Escano who inhibited himself from the case for being resulting from their bombardment of prejudicial publicity.
employed with NBI before. His pair Judge Escano issued warrant of arrest to
defendants. The case was re-raffled to Branch 274, presided by Judge Amelita Alvarez vs. CFI
Tolentino who issued new warrants of arrest. Webb and the others voluntarily
64 Phil. 33 (1937)
surrendered. They files before the court petition of certiorari, prohibition and
mandamus. They contend that (1) respondent Judges de Leon and Tolentino ARRESTS, SEARCHES AND SEIZURES > Examination of witnesses
gravely abused their discretion when they failed to conduct a preliminary
examination before issuing warrants of arrest against them: (2) the DOJ Panel Facts: The Anti-Usury Board of the Department of Justice presented to Judge
likewise gravely abused its discretion in holding that there is probable cause David a sworn affidavit that a certain Narciso Alvarez is in possession of
to charge them with the crime of rape with homicide; (3) the DOJ Panel denied books, receipts, chits, lists used by him as money lender/usurer charging
them their constitutional right to due process during their preliminary usurious rates in violation of law. Affiant Almeda, chief of the task force,
investigation; and (4) the DOJ Panel unlawfully intruded into judicial didn’t say that the information was based on his personal knowledge but was
prerogative when it failed to charge Jessica Alfaro in the Information as an only received by him from a reliable source. Subsequently, the judge issued
accused. the warrant ordering the search of Alvarez’ house. On June 4, 1936, the agents
raided the subject place and seized different documents namely, banknotes,
Issue: Whether or not the attendant publicity deprived Webb and the others of bankbooks, stubs, cashbooks, bills of lading, credit receipts, etc. Thereafter,
their right to fair trial? the articles seized was not brought immediately to the custody of the judge
who issued the SW. Alvarez moved that the agents of the Board be declared
Decision: Petition dismissed. to warrant a finding of prejudicial publicity there
guilty of contempt and prays that all articles in question be returned to him
must be allegation and proof that the judges have been unduly influenced, not
because the SW issued was illegal. On the other hand, the Anti-Usury Board
simply that they might be, by the barrage of publicity. In the case at bar, we
pleaded that they be allowed to retain custody of the articles seized for further
find nothing in the records that will prove that the tone and content, of the
investigation. When the judge sustained the latter’s motion. Alvarez elevated
publicity that attended the investigation of petitioners fatally infected the
the matter to the SC and prayed that the search warrant as well as the order of
fairness and impartiality of the DOJ Panel. Petitioners cannot just rely on the
the judge authorizing the Anti-Usury Board to retain custody be declared null
subliminal effects of publicity on the sense of fairness of the DOJ Panel, for
and void.
these are basically unbeknown and beyond knowing. To be sure, the DOJ
Panel is composed of an Assistant Chief State Prosecutor and Senior State Issue: Whether the SW issued by the judge is illegal for having solely as basis
Prosecutors. Their long experience in criminal investigation is a factor to the affidavit of Agent Almeda in whose oath the latter declared that he had no
consider in determining whether they can easily be blinded by the klieg lights personal knowledge of the facts which were to serve as basis for the issuance
of publicity. Indeed, their 26-page Resolution carries no indubitable indicia of of the warrant but he had knowledge thereof only through information secured
bias for it does not appear that they considered any extra-record evidence from a person whom he considered reliable.
except evidence properly adduced by the parties. The length of time the
investigation was conducted despite its summary nature and the generosity Ruling: Section 1, paragraph 3, of Article III of the Constitution and Section
with which they accommodated the discovery motions of petitioners speak 97 of General Orders 58 require that there be not only probable cause before
well of their fairness. At no instance, we note, did petitioners seek the
Page 20 of 33

judge is satisfied that there exists probable cause; when the applicant’s
the issuance of a search warrant but that the search warrant must be based upon knowledge of the facts is mere hearsay, the affidavit of one or more witnesses
an application supported by oath of the applicant and the witnesses he may having a personal knowledge of the facts is necessary. Thus the warrant issued
produce. In its broadest sense, an oath includes any form of attestation by is likewise illegal because it was based only on the affidavit of the agent who
which a party signifies that he is bound in conscience to perform an act had no personal knowledge of the facts
faithfully and truthfully; and it is sometimes defined as an outward pledge
given by the person taking it that his attestation or promise is made under an
immediate sense of his responsibility to God. The oath required must refer
to the truth of the facts within the personal knowledge of the petitioner or
his witnesses, because the purpose thereof is to convince the committing
magistrate, not the individual making the affidavit and seeking the
issuance of the warrant, of the existence of probable cause. The true test
of sufficiency of an affidavit to warrant issuance of a search warrant is
Luna v. Plaza
whether it has been drawn in such a manner that perjury could be
charged thereon and affiant be held liable for damages caused. The Facts:
affidavit, which served as the exclusive basis of the search warrant, is
insufficient and fatally defective by reason of the manner in which the oath The criminal action was commenced by T-Sgt. Candido Patosa, PC
was made, and therefore, the search warrant and the subsequent seizure of the investigator of Tandag, Surigao del Sur, by filing with respondent Municipal
books, documents and other papers are illegal. Further, it is the practice in this Judge Lorenzo M. Plaza, of the Municipal Court of Tandag, Criminal Case
jurisdiction to attach the affidavit of at least the applicant or complainant to No. 1138 charging the accused, herein petitioner, with the crime of murder.
the application. It is admitted that the judge who issued the search warrant in Supporting the complaint were sworn statements of the witnesses for the
this case, relied exclusively upon the affidavit made by agent Almeda and that prosecution, in the form of questions and answers taken by T-Sgt. Patosa, and
he did not require nor take the deposition of any other witness. The subscribed and sworn to before the respondent Judge at the time of the filing
Constitution does not provide that it is of an imperative necessity to take the of the complaint. The respondent Judge examined the prosecution witnesses
depositions of the witnesses to be presented by the applicant or complainant in by reading to them "all over again the questions and answers" in their
addition to the affidavit of the latter. The purpose of both in requiring the statements in writing, and the witnesses-affiants declared before said Judge
presentation of depositions is nothing more than to satisfy the committing that the questions were propounded by T-Sgt. Candido Patosa, and that the
magistrate of the existence of probable cause. Therefore, if the affidavit of the answers were made by them. The affiants further declared before respondent
applicant or complainant is sufficient, the judge may dispense with that of Judge that their answers were true, and were freely and voluntarily made; that
other witnesses. Inasmuch as the affidavit of the agent was insufficient because they fully understood the questions and answers, and that they were willing to
his knowledge of the facts was not personal but merely hearsay, it is the duty sign their respective affidavits. The affiants signed their respective affidavits
of the judge to require the affidavit of one or more witnesses for the purpose in the presence of the respondent Judge, who also signed after the usual
of determining the existence of probable cause to warrant the issuance of the procedure of administering the oath.
search warrant. When the affidavit of the applicant or complainant contains
sufficient facts within his personal and direct knowledge, it is sufficient if the
Page 21 of 33

All the conditions, in the afore-quoted Section 4, set forth to deny the writ, are
Considering the answers of the affiants to the questions contained in their present in the instant case. It is shown that petitioner is detained and is in the
sworn statements, together with the post-mortem and autopsy report on the custody of the respondent Provincial Warden by virtue of the order of arrest
dead body of the victim Jaime Diaz Ng, the certificate of death, the sketch dated February 18, 1967, and the order dated February 21, 1967, of respondent
showing the position of the victim and the accused, and Exhibits 6, 7, 8, 12, Judge, to confine petitioner in the provincial jail. It is not disputed by petitioner
and 13 of herein respondents, the respondent Judge opined that there was that respondent Judge had jurisdiction to issue the warrant of arrest and the
reasonable ground to believe that the crime of murder had been committed and order of commitment under the provisions of Section 47, Republic Act No.
the accused was probably guilty thereof. Respondent Judge issued the order 409, as amended by Republic Act No. 1201, although petitioner did question
and warrant of arrest, specifying therein that no bail should be accepted for the the validity of the warrant of arrest for allegedly having been issued in
provisional release of the accused. On February 20, 1967, upon motion of violation of Republic Act No. 3828 — which claim We have found to be
petitioner that he be admitted to bail upon the ground that the evidence of guilt untenable. Consequently, the trial Judge did not commit an error in denying
was not strong, respondent Judge issued an order granting bail, fixing it at the writ of habeas corpus prayed for.
P30,000.00; which order, however, respondent Judge later revoked, and
petitioner was denied bail. At any rate, We believe that, if at all, the remedy available to the petitioner
herein, under the circumstances stated in this opinion, is not a petition for a
Issue : writ of habeas carpus but a petition to quash the warrant of arrest or a petition
WON the petitioner was violated of his substantial rights during his for a reinvestigation of the case by the respondent Municipal Judge or by the
preliminary investigation. Provincial Fiscal.

Ruling:

We find that the trial Judge committed no error when he held that, based upon
the facts shown during the hearing of this case, respondent Municipal Judge
had substantially complied with the requirements of the law - specifically
Republic Act 3828 - before issuing the warrant of arrest in this case.

In the light of what has been said above, it appears clear that petitioner’s Marinas v. Siochi
second assignment of error, that the trial court erred in denying the writ
of habeas corpus, is untenable. Moreover, Section 4 of Rule 102 of the Rules Facts:
of Court provides in part, as follows:
The present controversy arose out of the issuance by the Municipal Court of
"SECTION. 4 When writ not allowed or discharge authorized. — If it appears
Pasig, Rizal, of a Writ of Execution in Civil Case No. 938 for Ejectment,
that the person alleged to be restrained of his liberty is in the custody of an
entitled Jose C. Zulueta vs. Gregorio Atienza. On December 13, 1965,
officer under process issued by a court or judge .. and that the court or judge
petitioner Antonio Marinas, Deputy Sheriff of Rizal, with his co-petitioners
had jurisdiction to issue the process .. or make the order, the writ shall not be
Antonio Montano and Gregorio Rupisan enforced said Writ of Execution by
allowed . . ."
Page 22 of 33

Complaints were filed by Lt. Jose S. Lontoc, Chief of the Criminal


levying upon the personal properties and chattels of private respondents Investigation Section of the Police Department of Pasig, Rizal, for and on
Victoria Lasin Vda. de Atienza and] Rosario L. Atienza, and taking out said behalf of the Chief of Police. These Complaints contained an annotation on
properties from their (respondents') rented house at #23 General Malvar St., the lower left hand corner reading: "APPROVED AFTER PRELIMINARY
Antonio Village, Pasig, Rizal. Respondents were also ejected from said house. EXAMINATION: (SGD) Lucila P. Alcoba, Special Counsel." The
On the same date, respondent Victoria Lasin Vda, de Atienza reported to the Complaints in Criminal Cases Nos. 12943 and 12944 for Theft and Grave
police authorities of Pasig that her jewelry worth P590.00 had been taken by Coercion, respectively, were subscribed and sworn to by Lt. Jose S. Lontoc
petitioners without issuing any receipt therefor, and in connection therewith, before respondent Judge. The Complaint in Criminal Case No. 12945 for Theft
she executed a written Statement which was sworn to before Special Counsel does not show the jurat on its face, but respondents state that it was also
Lucila P. Alcoba. attested to by Lt. Lontoc before respondent Judge and that this appears on the
dorsal side of the Complaint. On February 8, 1966, warrants for the arrest of
On January 28, 1966, respondents re-entered the house they had been ejected
petitioners were issued by respondent Judge in all three cases after preliminary
from after securing a Court Order for that purpose. Respondent Rosario L.
examination conducted by him in Criminal Cases Nos. 12943 and 12944, and
Atienza then discovered that several pieces of her jewelry and other personal
by Special Counsel Lucila P. Alcoba in Criminal Case No. 12945.
items, with a total value of P1,018.00, were missing. She reported the loss to
the authorities on February 2, 1966, and her Statement was taken. She
subscribed and swore to the same before respondent Municipal Judge Andres
S. Siochi. Petitioners took exception to the issuance of the warrants of arrest against them
and instituted the present Petition raising the following issues: 1äwphï1.ñët
On February 3, 1966, respondents, armed with a Court Order authorizing them
to enter the premises of the said house, did so again to get their remaining 1. When Section 87, Republic Act No. 296, as amended by Republic Act No.
unlevied properties. They claimed, however, that on the said date petitioners 3828, provides that when the penalty provided by law does not exceed prision
and their companions forcibly compelled them to deliver the unlevied personal correccional, then the Municipal Judge in the capitals of the provinces shall
properties found therein, hauled said articles into a truck and left. Private have "like jurisdiction as the Court of First Instance" to try the offense, does
respondents reported the incident to the police authorities at Pasig. Victoria the Municipal Court in such cases follow the procedure for Municipal Courts
Lasin executed a Statement alleging that the personal properties forcibly taken or that for Courts of First Instance?
from them by petitioners, amounting to P2,645.00, were not included in the
2. Is preliminary investigation a part of due process?
levy. Her son, Tranquilino Atienza, also executed an Affidavit corroborating
her declaration. 6Both Statements were subscribed and sworn to before 3. Can there be due process without the presence of the accused during the
respondent Judge. preliminary investigation.

On February 7, 1966, two separate charges for Theft, docketed as Criminal


Cases Nos. 12943 and 12945, were filed against petitioners and Carlos
Quintana before the Municipal Court of Pasig, Rizal, respondent Judge, Issue:
presiding. A Complaint for Grave Coercion Crime Case No. 12944) was also
lodged against petitioners and three Does on the same date. The three
Page 23 of 33

to the jurisdiction of the Court but merely to the regularity of the proceedings,
WON the petition should be granted? and bearing in mind that preliminary investigation can be waived, as in fact, it
is frequently waived.
Ruling:

While section l (3) Art. III of the 1935 Constitution does require, before the
issuance of a warrant of arrest, the determination of probable cause by the
Judge after examination of witnesses he may produce, the curtailment of the
presence of an accused during that preliminary examination entails no Marinas vs. Siochi
infringement of the constitutional right to due process of law nor to equal
Facts:
protection of the laws.
The present controversy arose out of the issuance by the MTC, of a Writ of
The loss of time entailed in the conduct of preliminary investigations, with the
Execution in a Civil Case for Ejectment, entitled Jose C. Zulueta vs.
consequent extension of deprivation of the accused's liberty, in case he fails to
Gregorio Atienza. Petitioner Antonio Marinas, Deputy Sheriff of Rizal,
post bail, which at times out- lasts the period of the penalty provided by law
with his co-petitioners enforced said Writ of Execution by levying upon
for the offense, besides the mental anguish suffered in protracted litigations,
the personal properties and chattels of private respondents Victoria Lasin Vda.
are eliminated with the assurance of a speedy and expeditious trial for the
de Atienza and Rosario L. Atienza, and taking outsaid properties from their
accused, upon his arraignment (without having to undergo the second stage of
rented house. Respondents were also ejected from said house. Respondent
the preliminary investigation), and of a prompt verdict on his guilt or
Victoria Lasin Vda, de Atienza reported to the police authorities of Pasig that
innocence. On the other hand, the so-called first stage of preliminary
her jewelry had been taken by petitioners without issuing any receiptand
investigation or the preliminary examination, conducted by the duly
she executed a written Statement which was sworn to before Special
authorized officer, as borne out by the examination and sworn written
Counsel Lucila P. Alcoba.Later, respondents re-entered the house they
statements of the complainants and their witnesses, generally offices to
had been ejected from after securing a Court Order and discovered that
establish the existence of reasonable ground to charge the accused with having
several pieces of her jewelry and other personal items were missing. She
committed the offense complained of.
reported the loss to the authorities and subscribed and swore to the same before
Attention should also be called to the fact that neither the 1935 nor the 1973 respondent Municipal Judge Andres S. Siochi. Respondents,then, entered their
Constitution requires the holding of a preliminary investigation. lt is settled house again to get their remaining unlevied properties. They claimed,
doctrine that the right hereto is of statutory character and may be invoked only however, that petitioners and their companions forcibly compelled them
when specifically created by statute. lt is not a fundamental right and may be to deliver the unlevied personal properties found therein, hauled said
waived expressly or by silence. articles into a truck and left. Private respondents reported the incident
to the police authorities. On February 7, 1966, two separate charges for
In a nutshell, the proceedings in these three criminal cases conformed to law Theft were filed against petitioners and before the MTC where respondent
and jurisprudence. But even conceding that petitioners were entitled to a Judge is presiding.A Complaint for Grave Coercion was also lodged
preliminary investigation, the proper forum before which absence thereof against petitioners. Warrants for the arrest of petitioners were issued by
should have been raised and ventilated was in the trial Court, not in an respondent Judge in all three cases after preliminary examination conducted
appellate Court because the absence of preliminary investigation does not go
Page 24 of 33

complying to the requirements of Section 87 of the Judiciary Act of 1948, as


by him in Criminal Cases Nos. 12943 and 12944, and by Special Counsel amended, providing therein that 'no warrant of arrest shall be issued by any
Lucila P. Alcoba in Criminal Case No. 12945. justice of the peace in any criminal case filed with him unless he first examines
the witness or witnesses personally, and the examination shall be under
Issue: oath and reduced to writing in the form of searching questions and
Whether or notthe right of the accused to meet the witnesses face to answers. The term "searching questions and answers" means only, taking
face is violated when respondent judge issued the warrant of arrest. into consideration the purpose of the preliminary examination which is to
determine 'whether there is a reasonable ground to believe that an offense has
Held: been committed and the accused is probably guilty thereof so that a warrant of
arrest may be issued and the accused held for trial', such questions as have
No. What was conducted by the respondent Judge in these cases is the tendency to show the commission of a crime and the perpetrator thereof. What
preliminary examination before the issuance of a warrant of arrest pursuant would be searching questions would depend on what is sought to be inquired
to section 1, Rule 112. The 1935 Constitution, in section l (3), Article into, such as: the nature of the offense, the date, time, and place of its
III provides that no warrant shall be issued but upon probable cause to commission, the possible motives for its commission the subject, his age,
be determined by the Judge after examination of witnesses under oath education, status, financial and social circumstances, his attitude toward the
or affirmation of the complaint and the witnesses he may produce. investigation, social attitudes, opportunities to commit the offense; the victim,
Conformably thereto, Section 87, paragraph 3, of the Judiciary Act, as his age, status, family responsibilities, financial and social circumstances,
amended by Republic Act No. 3828, provides that: before a Municipal Judge characteristics, etc. The points that are the subject of inquiry may differ from
may issue a warrant of arrest, the following conditions must first be case to case. The questions, therefore, must to a great degree depend upon
fulfilled: (1) he must examine the witness or witnesses personally; (2) the the Judge making the investigation. At any rate, the court a quo found
examination must be under oath; and (3) the examination must be reduced to that respondent Judge was satisfied that the questions and answers contained
writing in the form of searching questions and answers. These requirements in the sworn statements taken by T-Sgt. Patosa partake of the nature of his
have been met in the three criminal cases involved herein. Before the searching questions and answers as required by law,' so the respondent Judge
warrants of arrest were issued by the respondent Judge in Criminal Cases Nos. adopted them. InDe Mulata vs. Irizari,61 SCRA 210, 213 (1974):The
12944 and 12945, he first conducted the necessary preliminary examination requirement that the investigating judge must examine the witnesses
required by Section l of Rule 112 by adopting, as his own questions, personally, which examination shall be under oath and reduced to writing in
and by asking the complainants and their witnesses, the same or Identical the form of searching questions and answers, is fulfilled where the municipal
questions asked of them by the Investigating Police Officer in their judge examined under oath the witnesses by asking questions that were
written statements before the said Police Investigator and thereafter the adopted from a previous investigation and considered by him as sufficiently
respondent Judge required them (the complainants and their witnesses) to searching and, which questions and the answers thereto, were in writing
subscribe before and make oath to him as to the truth of the answers and sworn to before him prior to his issuance of the order of arrest.
given by them to the Police Investigator as shown by the fact that in
said Annexes, the deponents signed their respective names twice, once
before the Investigating Police Officer and the second time before the
respondent Judge who also required them to take the jurat to the oath, thereby Roan v. Gonzales, 145 SCRA 687 (1986)
Page 25 of 33

depositions in writing and attaching them to the record, rendering the search
warrant invalid. (See Rule 126, Sec 4)

FACTS: The challenged search warrant was issued by the respondent judge on The respondent judge also declared that he "saw no need to have applicant
May 10, 1984. The petitioner's house was searched two days later but none of Quillosa's deposition taken considering that he was applying for a search
the articles listed in the warrant was discovered. However, the officers warrant on the basis of the information provided by the witnesses whose
conducting the search found in the premises one Colt Magnum revolver and depositions had already been taken by the undersigned.
eighteen live bullets which they confiscated. They are now the bases of the
charge against the petitioner.

Respondent Judge said that when PC Capt. Mauro P. Quinosa personally filed Mata vs Bayona, G.R. No. L-50720, 26 March 1984
his application for a search warrant on May 10, 1984, he appeared before him
in the company of his two (2) witnesses, Esmael Morada and Jesus Tohilida,
both of whom likewise presented to him their respective affidavits taken by FACTS: Soriano Mata was accused under Presidential Decree (PD) 810, as
Pat. Josue V. Lining, a police investigator. As the application was not yet amended by PD 1306, the information against him alleging that Soriano Mata
subscribed and sworn to, he proceeded to examine Captain Quillosa on the offered, took and arranged bets on the Jai Alai game by “selling illegal tickets
contents thereof to ascertain, among others, if he knew and understood the known as ‘Masiao tickets’ without any authority from the Philippine Jai Alai
same. Afterwards, he subscribed and swore to the same before him. & Amusement Corporation or from the government authorities concerned.”
Mata claimed that during the hearing of the case, he discovered that nowhere
from the records of the said case could be found the search warrant and other
ISSUE: Whether the Respondent Judge failed to comply with the proper pertinent papers connected to the issuance of the same, so that he had to inquire
procedure in issuing the Search Warrant. from the City Fiscal its whereabouts, and to which inquiry Judge Josephine K.
Bayona, presiding Judge of the City Court of Ormoc replied, “it is with the
court”. The Judge then handed the records to the Fiscal who attached them to
the records. This led Mata to file a motion to quash and annul the search
HELD: Yes, mere affidavits of the complainant and his witnesses are thus not
warrant and for the return of the articles seized, citing and invoking, among
sufficient. The examining Judge has to take depositions in writing of the
others, Section 4 of Rule 126 of the Revised Rules of Court. The motion was
complainant and the witnesses he may produce and attach them to the record.
denied by the Judge on 1 March 1979, stating that the court has made a
Such written deposition is necessary in order that the Judge may be able to
thorough investigation and examination under oath of Bernardo U. Goles and
properly determine the existence or non-existence of the probable cause, to
Reynaldo T. Mayote, members of the Intelligence Section of 352nd PC
hold liable for perjury the person giving it if it will be found later that his
Co./Police District II INP; that in fact the court made a certification to that
declarations are false.
effect; and that the fact that documents relating to the search warrant were not
We, therefore, hold that the search warrant is tainted with illegality by the attached immediately to the record of the criminal case is of no moment,
failure of the Judge to conform with the essential requisites of taking the considering that the rule does not specify when these documents are to be
attached to the records. Mata’s motion for reconsideration of the aforesaid
Page 26 of 33

Facts:Respondent RTC Judge Esteban Lising, upon application filed by Lt.


order having been denied, he came to the Supreme Court, with the petition for Col. Berlin Castillo of the Philippine Constabulary Criminal Investigation
certiorari, praying, among others, that the Court declare the search warrant to Service, issued a search warrant authorizing the search and seizure of articles
be invalid for its alleged failure to comply with the requisites of the allegedly used by petitioner in committing the crime of inciting to sedition.
Constitution and the Rules of Court, and that all the articles confiscated under
such warrant as inadmissible as evidence in the case, or in any proceedings on 1. Printed copies of Philippine Times;
the matter.
2. Manuscripts/drafts of articles for publication in the Philippine Times;
ISSUE: WON the judge must before issuing the warrant personally examine
3. Newspaper dummies of the Philippine Times;
on oath or affirmation the complainant and any witnesses he may produce and
take their depositions in writing, and attach them to the record, in addition to 4. Subversive documents, articles, printed matters, handbills, leaflets, banners;
any affidavits presented to him?
5. Typewriters, duplicating machines, mimeographing and tape recording
HELD:YES. Under the Constitution “no search warrant shall issue but upon machines, video machines and tapes
probable cause to be determined by the Judge or such other responsible officer
as may be authorized by law after examination under oath or affirmation of the Petitioner filed an urgent motion to recall warrant and to return
complainant and the witnesses he may produce”. More emphatic and detailed documents/personal properties alleging among others that the properties
is the implementing rule of the constitutional injunction, The Rules provide seized are typewriters, duplicating machines, mimeographing and tape
that the judge must before issuing the warrant personally examine on oath or recording machines, video machines and tapes which are not in any way,
affirmation the complainant and any witnesses he may produce and take their inanimate or mute things as they are, connected with the offense of inciting to
depositions in writing, and attach them to the record, in addition to any sedition.
affidavits presented to him. Mere affidavits of the complainant and his
Respondent Judge Lising denied the motion. Hence, this petition praying that
witnesses are thus not sufficient. The examining Judge has to take depositions
the search warrant issued by respondent Judge Esteban M. Lising be declared
in writing of the complainant and the witnesses he may produce and to attach
null and void ab initio that the padlocked office premises of the Philippine
them to the record. Such written deposition is necessary in order that the Judge
Times be reopened.
may be able to properly determine the existence or nonexistence of the
probable cause, to hold liable for perjury the person giving it if it will be found Respondents would have this Court dismiss the petition stating that probable
later that his declarations are false. We, therefore, hold that the search warrant cause exists justifying the issuance of a search warrant, the articles seized were
is tainted with illegality by the failure of the Judge to conform with the adequately described in the search warrant, a search was conducted I n an
essential requisites of taking the depositions in writing and attaching them to orderly manner and the padlocking of the searched premises was with the
the record, rendering the search warrant invalid. consent of petitioner's wife.

Issue: WON the search warrant valid.


Corro vs. Lising, 137 SCRA 541 Held:
Page 27 of 33

The affidavit of Col. Castillo states that in several issues of the Philippine
Section 3, Article IV of the 1973 Constitution provides: Times:

SEC. 3. ...no search warrant or warrant of arrest issue except upon probable ... we found that the said publication in fact foments distrust and hatred against
cause to be determined by the judge, or such other responsible officer as may the government of the Philippines and its duly constituted authorities, defined
be authorized by law, after examination under oath or affirmation of the and penalized by Article 142 of the Revised Penal Code as amended by
complainant and the witnesses he may produce, and particularly describing the Presidential Decree No. 1835; (p. 22, Rollo)
place to be searched and the persons or things to be seized.
and, the affidavit of Lt. Ignacio reads, among others—
and, Section 3, Rule 126 of the New Rules of Court, states that:
... the said periodical published by Rommel Corro, contains articles tending to
SEC. 3. Requisites for issuing search warrant. — A search warrant shall not incite distrust and hatred for the Government of the Philippines or any of its
issue but upon probable cause in connection with one specific offense to be duly constituted authorities. (p. 23, Rollo)
determined by the judge or justice of the peace after examination under oath
or affirmation of the complainant and the witnesses he may produce, and The above statements are mere conclusions of law and will not satisfy the
particularly describing the place to be searched and the persons or things to be requirements of probable cause. They cannot serve as basis for the issuance of
seized. search warrant, absent of the existence of probable cause.

Probable cause may be defined as "such reasons, supported by facts and In the case at bar, the search warrant issued by respondent judge allowed
circumstances, as will warrant a cautious man in the belief that his actions, and seizure of printed copies of the Philippine Times, manuscripts/drafts of articles
the means taken in prosecuting it, are legally just and proper (Burton vs. St. for publication, newspaper dummies, subversive documents, articles, etc., and
Paul, M & M. Ry. Co., 33 Minn. 189, cited in U.S. vs. Addison, 28 Phil. 566)." even typewriters, duplicating machines, mimeographing and tape recording
machines.
An application for search warrant must state with particularly the alleged
subversive materials published or intended to be published by the publisher Thus, the language used is so all embracing as to include all conceivable
and editor of the Philippine Times, Rommel Corro. As We have stated in records and equipment of petitioner regardless of whether they are legal or
Burgos, Sr. vs. Chief of Staff of the Armed Forces of the Philippines, 133 illegal. The search warrant under consideration was in the nature of a general
SCRA 800, "mere generalization will not suffice." warrant which is constitutionally objectionable.

A search warrant should particularly describe the place to be searched and the
things to be seized. "The evident purpose and intent of this requirement is to
Nolasco vs. Cruz Pano, 132 SCRA 152 (1985)
limit the things to be seized to those, and only those, particularly described in
the search warrant- to leave the officers of the law with no discretion regarding
what articles they should seize, to the end that unreasonable searches and
seizures may not be committed, — that abuses may not be committed. FACTS: Milagros Aguilar-Roque was arrested together with Cynthia Nolasco
by the Constabulary Security Group (CSG). Milagros had been wanted as a
Page 28 of 33

guideline to the searching team as to what items might be lawfully seized thus
high ranking officer of the CPP. The arrest took place at 11:30 a.m. of August giving the officers of the law discretion regarding what articles they should
6, 1984. At noon of the same day, her premises were searched and 428 seize as, in fact, taken also were a portable typewriter and 2 wooden boxes.
documents, a portable typewriter and 2 boxes were seized.
It is thus in the nature of a general warrant and infringes on the constitutional
Earlier that day, Judge Cruz Paño issued a search warrant to be served at mandate requiring particular description of the things to be seized. In the recent
Aguilar-Roque’s leased residence allegedly an underground house of the rulings of this Court, search warrants of similar description were considered
CPP/NPA. On the basis of the documents seized, charges of subversion and null and void for being too general.
rebellion by the CSG were filed by but the fiscal’s office merely charged her
and Nolasco with illegal possession of subversive materials. Aguilar-Roque
asked for suppression of the evidence on the ground that it was illegally
Burgos vs Chief of Staff, G.R. No. L-64261, December 26, 1984
obtained and that the search warrant is void because it is a general warrant
since it does not sufficiently describe with particularity the things subject of
the search and seizure, and that probable cause has not been properly
established for lack of searching questions propounded to the applicant’s Facts: Petitioners question the issuance of the two search warrants on the
witness. premises of “Metropolitan Mail” and “We Forum” newspapers and the seizure
of office and printing machines, equipment, paraphernalia, motor vehicles and
ISSUE: WON the search warrant was valid? other articles used in the printing, publication and distribution of the said
newspapers, as well as numerous papers, documents, books and other written
HELD:
literature alleged to have been used in subversive activities.
NO. Section 3, Article IV of the Constitution, guarantees the right of the people
It is contended by petitioners that the documents necessitating the issuance of
to be secure in their persons, houses, papers and effects against unreasonable
the subject warrants could not have provided sufficient basis for the finding of
searches and seizures of whatever nature and for any purpose. It also
a probable cause upon which a warrant may validly issue in accordance with
specifically provides that no Search Warrant shall issue except upon probable
Section 3, Article IV of the 1973 Constitution which provides:
cause to be determined by the Judge or such other responsible officer as may
be authorized by law, after examination under oath or affirmation of the “SEC. 3. x x x and no search warrant or warrant of arrest shall issue except
complainant and the witnesses he may produce, and particularly describing the upon probable cause to be determined by the judge, or such other responsible
place to be searched and the things to be seized. officer as may be authorized by law, after examination under oath or
affirmation of the complainant and the witnesses he may produce, and
It is at once evident that the foregoing Search Warrant authorizes the seizure
particularly describing the place to be searched and the persons or things to be
of personal properties vaguely described and not particularized. It is an all-
seized.”
embracing description which includes everything conceivable regarding the
Communist Party of the Philippines and the National Democratic Front. It does Issue: Whether the warrants were validly issued
not specify what the subversive books and instructions are; what the manuals
not otherwise available to the public contain to make them subversive or to Held: No.
enable them to be used for the crime of rebellion. There is absent a definite
Page 29 of 33

witnesses, because the purpose thereof is to convince the committing


Probable cause for a search is defined as such facts and circumstances which magistrate, not the individual making the affidavit and seeking the issuance of
would lead a reasonably discreet and prudent man to believe that an offense the warrant, of the existence of probable cause.” As couched, the quoted
has been committed and that the objects sought in connection with the offense averment in said joint affidavit filed before respondent judge hardly meets the
are in the place sought to be searched. And when the search warrant applied test of sufficiency established by this Court in Alvarez case.
for is directed against a newspaper publisher or editor in connection with the
publication of subversive materials, as in the case at bar, the application and/or Another factor which makes the search warrants under consideration
its supporting affidavits must contain a specification, stating with particularity constitutionally objectionable is that they are in the nature of general warrants.
the alleged subversive material he has published or is intending to publish.
Mere generalization will not suffice. Thus, the broad statement in Col.
Abadilla’s application that petitioner “is in possession or has in his control People vs Burgos
printing equipment and other paraphernalia, news publications and other
documents which were used and are all continuously being used as a means of September 14, 1986
committing the offense of subversion punishable under Presidential Decree
885, as amended x x x” is a mere conclusion of law and does not satisfy the
requirements of probable cause. Bereft of such particulars as would justify a Facts: Defendant is charged with illegal possession of firearm in furtherance
rinding of the existence of probable cause, said allegation cannot serve as basis of subversion (tasks such as recruiting members to the NPA and collection of
for the issuance of a search warrant and it was a grave error for respondent contributions from its members) and found guilty by the RTC of Digos, Davao
judge to have done so. del Sur. From the information filed by the police authorities upon the
information given by Masamlok, allegedly a man defendant tried to recruit into
Equally insufficient as basis for the determination of probable cause is the
the NPA, the police authorities arrest defendant and had his house searched.
statement contained in the joint affidavit of Alejandro M. Gutierrez and Pedro
Subsequently, certain NPA-related documents and a firearm, allegedly issued
U. Tango, “that the evidence gathered and collated by our unit clearly shows
and used by one Alias Cmdr. Pol of the NPA, are confiscated. Defendant
that the premises above- mentioned and the articles and things above-described
denies being involved in any subversive activities and claims that he has been
were used and are continuously being used for subversive activities in
tortured in order to accept ownership of subject firearm and that his alleged
conspiracy with, and to promote the objective of, illegal organizations such as
extrajudicial statements have been made only under fear, threat and
the Light-a-Fire Movement, Movement for Free Philippines, and April 6
intimidation on his person and his family. He avers that his arrest is unlawful
Movement.”
as it is done without valid warrant, that the trial court erred in holding the
In mandating that “no warrant shall issue except upon probable cause to be search warrant in his house for the firearm lawful, and that the trial court erred
determined by the judge, x x x after examination under oath or affirmation of in holding him guilty beyond reasonable doubt for violation of PD 9 in relation
the complainant and the witnesses he may produce; the Constitution requires to GOs 6and 7.
no less than personal knowledge by the complainant or his witnesses of the
facts upon which the issuance of a search warrant may be justified. In Alvarez
vs. Court of First Instance, this Court ruled that “the oath required must refer
to the truth of the facts within the personal knowledge of the petitioner or his
Page 30 of 33

not enough that there is reasonable ground to believe that the person to
Issue: If defendant’s arrest, the search of his home, and the subsequent be arrested has committed a crime in a warrantless arrest. An essential
confiscation of a firearm and several NPA-related documents are lawful. precondition is that a crime must have been in fact or actually have been
committed first; it isn’t enough to suspect a crime may have been
committed. The test of reasonable ground applies only to the identity of the
Held: Records disclose that when the police went to defendant’s house to arrest perpetrator. The Court also finds no compelling reason for the haste with
him upon the information given by Masamlok, they had neither search nor which the arresting officers sought to arrest the accused. We fail to see why
arrest warrant with them—in wanton violation of ArtIV, Sec 3 (now Art III, they failed to first go through the process of obtaining a warrant of arrest, if
sec 2). As the Court held in Villanueva vs Querubin, the state, however indeed they had reasonable ground to believe that the accused had truly
powerful, doesn’t have access to a man’s home, his haven of refuge where committed a crime. There is no showing that there was a real apprehension
his individuality can assert itself in his choice of welcome and in the kind that the accused was on the verge of flight or escape. Likewise, there is no
of objects he wants around him. In the traditional formulation, a man’s showing that the whereabouts of the accused were unknown.
house, however humble, is his castle, and thus is outlawed any unwarranted
intrusion by the government.
In proving the ownership of the questioned firearm and alleged subversive
The trial court justified the warrantless arrest under Rule 113 Sec 6 of the RoC:
documents, assuming they were really illegal, the defendant was never
a) When the person to be arrested has committed, is actually committing, or is informed of his constitutional rights at the time of his arrest; thus the
about to commit an offense in his presence; admissions obtained are in violation of the constitutional right against self-
incrimination under Sec 20 Art IV (now Sec 12, Art III) and thus inadmissible
b) When an offense has in fact been committed, and he has reasonable ground as evidence. Furthermore, the defendant was not accorded his constitutional
to believe that the person to be arrested has committed it; right to be assisted by counsel during the custodial interrogation. His extra-
judicial confession, the firearm, and the alleged subversive documents are all
c) When the person to be arrested is a prisoner who has escaped from a penal inadmissible as evidence. In light of the aforementioned, defendant is acquitted
establishment or place where he is serving final judgment or temporarily on grounds of reasonable doubt of the crime with which he has been charged.
confined while his case is pending or has escaped while being transferred from Subject firearm and alleged subversive documents have been disposed of in
one confinement to another and the confiscation of the firearm under Rule 126, accordance with law. The Court also maintains that violations of human rights
Sec 12: do not help in overcoming a rebellion. Reiterating Morales vs Enrile, “while
A person charged with an offense may be searched for dangerous weapons or the government should continue to repel the communists, the subversives,
anything which may be used as proof of the commission of the offense. the rebels, and the lawless with the means at its command, it should
always be remembered that whatever action is taken must always be
However, the trial court has erred in its conclusion that said warrantless within the framework of our Constitution and our laws.”
arrest is under the ambit of aforementioned RoC. At the time of defendant’s
arrest, he wasn’t in actual possession of any firearm or subversive document,
and was not committing any “subversive” act—he was plowing his field. It is
Page 31 of 33

that he only has two watches during that time and that he did not sufficiently
proved the injuries allegedly sustained.

PEOPLE VS AMMINUDIN Issue: Whether or not search of defendant’s bag is legal.

July 6, 1988

Held: The search was illegal. Defendant was not caught in flagrante delicto,
which could allow warrantless arrest or search. At the moment of his arrest, he
Facts: Idel Aminnudin, accused-appellant was arrested on June 25, 1984,
was not committing a crime. Nor was he about to do so or had just done so. To
shortly after disembarking from the M/V Wilcon 9 at about 8:30 in the
all appearances, he was like any of the other passengers innocently
evening, in Iloilo City. The PC officers who were in fact waiting for him
disembarking from the vessel. The said marijuana therefore could not be
because of a tip from one their informers simply accosted him, inspected his
appreciated as evidence against the defendant, and furthermore he is acquitted
bag and finding what looked like marijuana leaves took him to their
of the crime as charged.
headquarters for investigation. The two bundles of suspect articles were
confiscated from him and later taken to the NBI laboratory for examination. It
was found to contain three kilos of what were later analyzed as marijuana
leaves by an NBI forensic examiner. An information for violation of the
Dangerous Drugs Act was filed against him. Later, the information was
amended to include Farida Ali y Hassen, who had also been arrested with him
that same evening and likewise investigated. Both were arraigned and pleaded PONSICA VS IGNALAGA
not guilty. Subsequently, the fiscal filed a motion to dismiss the charge against
Ali on the basis of a sworn statement of the arresting officers absolving her July 3, 1987
after a 'thorough investigation." The motion was granted, and trial proceeded
only against the accused-appellant, who was eventually convicted. In his
defense, Aminnudin disclaimed the marijuana, averring that all he had in his Facts: The chief issue raised by the petitioners in this case is whether or not
bag was his clothing consisting of a jacket, two shirts and two pairs of pants. Section 143 of the Local Government Code1granting power to the municipal
He alleged that he was arbitrarily arrested and immediately handcuffed. His mayor to conduct preliminary investigations and order the arrest of the
bag was confiscated without a search warrant. At the PC headquarters, he was accused, was repealed by the 1985 Rules on Criminal Procedure promulgated
manhandled to force him to admit he was carrying the marijuana, the by this Court; and is, in addition, unconstitutional as vesting the power to
investigator hitting him with a piece of wood in the chest and arms even as he conduct preliminary investigations in an official who cannot be deemed a
parried the blows while he was still handcuffed. He insisted he did not even "neutral and detached magistrate" within the contemplation of Section 3,
know what marijuana looked like and that his business was selling watches Article IV of the 1973 Constitution. The issue is hereby resolved adversely to
and sometimes cigarettes. However the RTC rejected his allegations. Saying
Page 32 of 33

search warrants, arrested people without warrant of arrest, denied visitation


the petitioners, with the stressed qualification that the mayor's power to order rights, and interrogated them with the use of threats and tortures. A motion to
arrest ceased to exist as of February 2, 1987 when the new Constitution was dismiss was filed by defendants, stating that 1) plaintiffs may not cause a
ratified by the Filipino people, and that, in any event, the investigation actually judicial inquiry about their detention because the writ of habeas corpus was
conducted by respondent mayor in the case at bar was fatally defective. suspended; 2) defendants are immune from liability for acts done in their
official duties; 3) there was no cause of action. On Nov 8, 1983, Judge Fortun
granted the motion to dismiss, which prompted plaintiffs to file a MR on Nov
Ruling: While it is true that the mayors do "exercise general supervision over 18, 1983. He later inhibited himself and was replaced Judge Lising, who
units and elements of the INP stationed or assigned in their respective denied the MR for being filed out of time. Another MR was filed, and was
jurisdictions," they are not themselves directly involved in police work and only modified to include Maj. Aguinaldo and MSgt. Balaba for officers
cannot in any sense be described, as the petitioners do, as being deeply accountable in the said complaint.
involved in law enforcement functions. And even if that "deep involvement"
Issues:
be conceded, it does not follow that this would necessarily preclude their
assuming "the cold neutrality of an impartial judge" in conducting preliminary
investigations of persons suspected of crimes. As the law now stands, the
mayor may no longer conduct preliminary investigation, the authority to do so 1. Whether or not immunity from suit may be invoked?
being limited under Section 2, Rule 1 1 2 of the Rules of Court to (1) provincial
2. Whether petitioners have the right to question the alleged violation of their
or city fiscals and their assistants; (2) judges of the Municipal Trial Courts and
rights in the constitution?
Municipal Circuit Trial Courts; (3) national and regional state prosecutors; and
(d) such other officers as may be authorized by law. But only "the judge" may 3. Whether the superior officers who gave the orders are liable?
issue search and arrest warrants after due determination of probable cause.

Held:

1. NO, Article 32 of the Civil Code provides a sanction to rights and freedom
ABERCA v. VER enshrined in the constitution. These rights cannot be violated just because of
an order given by a superior. The rule of law must prevail, or else liberty will
April 15, 1988
perish.

Even though they just followed the orders of their superior, these do not
Facts: Task Force Makabansa (TFM) was ordered by General Fabian Ver to authorize them to disregard the rights of the petitioners, and therefore cannot
conduct preemptive strikes against Communist- Terrorist underground houses. be considered “acts done in their official duties”. Article 32 speaks of any
TFM raided several houses, employing in most cases defectively judicial
Page 33 of 33

public officer or private individual, and violation of these constitutional rights


does not exempt them from responsibility.

2. YES, the suspension of the writ of habeas corpus does not prevent
petitioners from claiming damages for the illegal arrest and detention in
violation of their constitutional rights by seeking judicial authority. What the
writ suspends is merely the right of an individual to seek release from detention
as a speedy means of obtaining liberty. It cannot suspend their rights and cause
of action for injuries suffered due to violation of their rights.

3. YES, Article 32 speaks of the liabilities of people who are in direct violation
of the rights stated, as well as people who are indirectly responsible for such
acts. In the case at hand, the superior officers are the ones who gave the order,
and can be considered indirectly responsible. It was also stated in the
complaint who were the ones who directly and indirectly participated in those
acts. By filing a motion to dismiss, they admitted all the facts stated in the
complaint.

Você também pode gostar